Computers and Technology
Computers and Technology, 27.01.2020 23:31, danleaz01

How do i take a pic on a hp laptop?

answer
Answers: 2

Other questions on the subject: Computers and Technology

image
Computers and Technology, 22.06.2019 17:00, juandguardado7045
Your company has 1,500 desktop computers running windows 7. you want to upgrade them to windows 10. which type of microsoft license would be best suited in this situation?
Answers: 3
image
Computers and Technology, 22.06.2019 17:00, shimmerandshine1
Aisha has finished working on a word processing document that contains 15 pages. she has added some special elements in the first three pages, page 9 and 10, and page 15 from the document. she wants to print only these pages to see how they look. which option is the correct way to represent (in the print dialog box) the pages that aisha wants to print
Answers: 3
image
Computers and Technology, 22.06.2019 20:00, Jana1517
What is the worst-case complexity of the maxrepeats function? assume that the longest string in the names array is at most 25 characters wide (i. e., string comparison can be treated as o( class namecounter { private: int* counts; int nc; string* names; int nn; public: namecounter (int ncounts, int nnames); int maxrepeats() const; }; int namecounter: : maxrepeats () { int maxcount = 0; for (int i = 0; i < nc; ++i) { int count = 1; for (int j = i+1; j < nc; ++j) { if (names[i] == names[j]) ++count; } maxcount = max(count, maxcount); } return maxcount; }
Answers: 3
image
Computers and Technology, 22.06.2019 23:50, Emptypockets451
You need to design a circuit that implements the functions in the following table: s0 s1 function0 0 a + 10 1 a – b1 0 a + b1 1 a – 1s0 and s1 are 1-bit control inputs to select the function of the circuit. inputs a and b are 4-bitnumbers in 2s complement form. the output is also a 4-bit number in 2s complement form. you are allowed to use only one ttl 7483 4-bit adder to implement all the functions. but anynumber of other components (except the adder) can be used. hint: design a combinational logic circuit to modify the input b and the β€œcarry input” of theadder depending on the control inputs s0 and s1.important: lab grade will depend on the working of the circuit & will be checked of by your labinstructor.1. is the output valid for the following input combinations: a. s0 = 0, s1 = 0, a = 7, b = 3? b. s0 = 0, s1 = 1, a = 7, b = 3? c. s0 = 1, s1 = 0, a = -4, b = -5? d. s0 = 1, s1 = 1, a = -8, b = 6? 2. what is the range of inputs (for both a and b) that will produce the valid output for all the functions?
Answers: 3
Do you know the correct answer?
How do i take a pic on a hp laptop?...

Questions in other subjects:

Konu
Social Studies, 06.01.2020 07:31
Konu
Mathematics, 06.01.2020 07:31